Statistics problem-exponential approximation

Click For Summary
The discussion revolves around calculating the probability P(X>k) for drawing balls from a box containing 2n balls of n different colors, focusing on finding a formula for k when P(X>k) is approximately 1/2 as n becomes large. For part a, the initial formula derived for P(X>k) was corrected to reflect the probability that the first k colors drawn are all different, leading to a more accurate representation involving fractions of n. In part b, participants discussed using exponential approximations and Stirling's approximation to simplify calculations for large n. The conversation emphasized the importance of correctly expressing the probabilities and understanding the underlying logic of the formulas used. The thread highlights key steps in deriving the necessary statistical formulas for the given problem.
nuagerose
Messages
12
Reaction score
0
Statistics problem---exponential approximation

Homework Statement


A box contains 2n balls of n different colors, with 2 of each color. Balls are picked at random from the box with replacement until two balls of the same color have appeared. Let X be the number of draws made.

a) Find a formula for P(X>k) k=2,3,...

b) Assuming n is large, use an exponential approximation to find a formula for k in terms of n such that P(X>k) is approximately 1/2. Evaluate k for n equal to one million.


Homework Equations





The Attempt at a Solution



For part a), I got that P(X>k) = (2n-2)/2n * (2n-4)/2n *...* (2n-2k+2n)/2n for k terms.

For part b), how do I set up an exponential approximation? To get started, I think that it would be e^(-1) + e^(-2) +...+ e^(1-k)... am I on the right track?
 
Physics news on Phys.org
nuagerose said:
For part a), I got that P(X>k) = (2n-2)/2n * (2n-4)/2n *...* (2n-2k+2n)/2n for k terms.

I haven't done the counting, but this formula looks wrong since it appears to have ##n-k## terms. Once you work out the right form, I would clean it up by canceling common factors of 2 and also using the factorial function.


For part b), how do I set up an exponential approximation? To get started, I think that it would be e^(-1) + e^(-2) +...+ e^(1-k)... am I on the right track?

They could mean use http://en.wikipedia.org/wiki/Stirling's_approximation]Stirling's[/PLAIN] approximation, which is useful whenever you have an expression for the factorial of a large number.
 
Last edited by a moderator:
nuagerose said:

Homework Statement


A box contains 2n balls of n different colors, with 2 of each color. Balls are picked at random from the box with replacement until two balls of the same color have appeared. Let X be the number of draws made.

a) Find a formula for P(X>k) k=2,3,...

b) Assuming n is large, use an exponential approximation to find a formula for k in terms of n such that P(X>k) is approximately 1/2. Evaluate k for n equal to one million.


Homework Equations





The Attempt at a Solution



For part a), I got that P(X>k) = (2n-2)/2n * (2n-4)/2n *...* (2n-2k+2n)/2n for k terms.

For part b), how do I set up an exponential approximation? To get started, I think that it would be e^(-1) + e^(-2) +...+ e^(1-k)... am I on the right track?

You really ought to explain the logic: your formula is the probability that the first k colours are all different. Of course, you can cancel out all the 2s to get
P(X > k) = \frac{n-1}{n} \frac{n-2}{n} \cdots \frac{n-k +1}{n}
(Note: the final factor is NOT what you wrote, but I assume that was just a 'typo', since you otherwise seemed to know what you were doing.)

You can do something similar to what Feller would do in his probability textbook, and re-write the result as
P(X &gt; k) = \left(1 - \frac{1}{n}\right)\left( 1 - \frac{2}{n}\right) \cdots <br /> \left( 1 - \frac{k-1}{n}\right)
That provides a convenient starting point.
 
Question: A clock's minute hand has length 4 and its hour hand has length 3. What is the distance between the tips at the moment when it is increasing most rapidly?(Putnam Exam Question) Answer: Making assumption that both the hands moves at constant angular velocities, the answer is ## \sqrt{7} .## But don't you think this assumption is somewhat doubtful and wrong?

Similar threads

  • · Replies 3 ·
Replies
3
Views
2K
  • · Replies 6 ·
Replies
6
Views
1K
  • · Replies 2 ·
Replies
2
Views
2K
  • · Replies 9 ·
Replies
9
Views
3K
  • · Replies 4 ·
Replies
4
Views
2K
Replies
1
Views
3K
  • · Replies 15 ·
Replies
15
Views
2K
  • · Replies 10 ·
Replies
10
Views
2K
Replies
1
Views
1K
  • · Replies 4 ·
Replies
4
Views
2K